LSAT and Law School Admissions Forum

Get expert LSAT preparation and law school admissions advice from PowerScore Test Preparation.

User avatar
 Dave Killoran
PowerScore Staff
  • PowerScore Staff
  • Posts: 5852
  • Joined: Mar 25, 2011
|
#87692
Complete Question Explanation
(The complete setup for this game can be found here: lsat/viewtopic.php?t=1469)

The correct answer choice is (E)

If M is one of the courses taken, then from the rules H, P, and T cannot be taken:

G4-Q21-d1.png

The only three remaining courses are L, S, and W. But, from the third rule, S and W cannot both be taken together, creating a hurdle the uncertainty situation:

G4-Q21-d2.png

Because S and W can only fill one of the two remaining courses that must be taken, regardless of which of S and W is taken, L must be taken. Consequently, answer choice (E) is correct.
You do not have the required permissions to view the files attached to this post.

Get the most out of your LSAT Prep Plus subscription.

Analyze and track your performance with our Testing and Analytics Package.